Convergence of Improper Integral: 1 / ( x^(1/3)*(/x-5/^(1/3))*(1 + sqrt(x))^0.7)

In summary, Mark showed that the following improper integral converges by comparing it to 1/x^(1/3) and 1//x-5/^(1/3), and that the last integral is divergent. He broke it up into three integrals and showed that the first two converged, but the last one did not. He found that the denominator is roughly comparable to x61/60 and that the integrand is less than 1/x61/60, so the integral converges.
  • #1
dsta
9
0
Establish convergence/divergence of the following improper integral:

integral from 0 to infinity of 1 / ( x^(1/3)*(/x-5/^(1/3))*(1 + sqrt(x))^0.7) )

My attempt at a solution was to break it up into 3 intergrals: 0 to 1, 1 to 5, and 5 to infinity...I showed that the first two of these integrals converges by comparing them to 1/x^(1/3) and 1//x-5/^(1/3) respectively. I can't establish convergence/divergence of last intergral from 5 to infinity, as hard as I have tried.

Thanks, any help is much appreciated.
 
Physics news on Phys.org
  • #2
dsta said:
Establish convergence/divergence of the following improper integral:

integral from 0 to infinity of 1 / ( x^(1/3)*(/x-5/^(1/3))*(1 + sqrt(x))^0.7) )

My attempt at a solution was to break it up into 3 intergrals: 0 to 1, 1 to 5, and 5 to infinity...I showed that the first two of these integrals converges by comparing them to 1/x^(1/3) and 1//x-5/^(1/3) respectively. I can't establish convergence/divergence of last intergral from 5 to infinity, as hard as I have tried.

Thanks, any help is much appreciated.

Is there some reason you wrote /x - 5/ using the slashes?
Why did you choose 1 as a limit of integration in two of your integrals? There is nothing unusual happening at x = 1.
In your integrals, what do you have for the integrands? Please show us the work you did.

BTW, there is no such word as "intergral."
 
  • #3
By /x-5/, I mean the absolute value of (x-5).

I chose 1 because don't you have to break up the original improper integral into appropriate intervals to see what's going on? There are unusual things happening at x=0 and x=5, and 1 was chosen to 'link up' these separate intervals (for want of a better phrase).

So I have this:
[0 to 1] 1 / ( x^(1/3)*(/x-5/^(1/3))*(1 + sqrt(x))^0.7) ) + [1 to 5]1 / ( x^(1/3)*(/x-5/^(1/3))*(1 + sqrt(x))^0.7) ) + [5 to infinity]1 / ( x^(1/3)*(/x-5/^(1/3))*(1 + sqrt(x))^0.7) )

The first two integrals I have shown to be convergent (by comparison), but I am stuck on the last one. Mabye I am going about this the wrong way altogether, but I'm sort of following a similar example from my lecture notes.

(Sorry about the messiness, I am trying to learn Latex)
 
  • #4
1+sqrt(x)>sqrt(x), and for large x, x-5>0.5x
 
  • #5
P.S. You also should break [5,infty) into two intervals.
 
  • #6
OK, I understand now why you are using 1 as a limit of integration. For absolute values, you can use this character: |.

To help you out with your LaTeX, your last integral is
[tex]\int_5^{\infty}\frac{1}{x^{1/3}|x - 5|^{1/3}(1 + \sqrt{x})^{0.7}}dx[/tex]
You'll need to break this into two integrals, say from a to 6 and from 6 to b, and look at the limit of each as a approaches 5 from above and as b approaches infinity. You can click on the integral and see the LaTeX code, and can copy and paste it to create your own.

Since in both integrals, x >= 5, you can drop the absolute value signs so you have (x^2 - 5x)^(1/3). I doubt that the integral can be found by ordinary means, so you'll probably need to use another comparison.

I'll need to think about this one...
 
  • #7
The denominator is roughly comparable with x61/60, with that exponent obtained by adding 2/3 and 7/20. Since this exponent is > 1, the integral of this function, from 6 to infinity, converges. If you can show that your denominator is > x61/60, and thereby that your integrand is < 1/x61/60, you're home free.
 
  • #8
Ahh okay, that makes a lot more sense now, thanks a lot Mark.

When I finish writing it up I will post my final explanation.
 

FAQ: Convergence of Improper Integral: 1 / ( x^(1/3)*(/x-5/^(1/3))*(1 + sqrt(x))^0.7)

1. What is an improper integral?

An improper integral is an integral where either the upper or lower limit of integration is infinite, or the function being integrated is undefined at one or more points within the integration interval.

2. How do you determine if an integral is improper?

An integral is considered improper if one or both of the following conditions are met: the integration interval is infinite, or the function being integrated is undefined at one or more points within the integration interval.

3. How do you evaluate an improper integral?

To evaluate an improper integral, you must first rewrite it as a limit of a proper integral. Then, you can use techniques such as integration by parts, substitution, or partial fractions to evaluate the limit and determine the value of the improper integral.

4. What is the difference between a convergent and a divergent improper integral?

A convergent improper integral is one where the limit of the proper integral exists and is a finite value. A divergent improper integral is one where the limit of the proper integral does not exist or is infinite.

5. Can improper integrals be solved using the Fundamental Theorem of Calculus?

Yes, improper integrals can be solved using the Fundamental Theorem of Calculus. However, in order to use this theorem, the improper integral must first be rewritten as a limit of a proper integral.

Similar threads

Replies
10
Views
1K
Replies
7
Views
1K
Replies
16
Views
3K
Replies
13
Views
2K
Replies
1
Views
907
Replies
3
Views
6K
Replies
5
Views
1K
Replies
1
Views
836
Back
Top